OBM 2015 - Nível 2 - P5

PROBLEMA 5

Seja n um inteiro positivo e sejam n = d_1  data-recalc-dims= d_2 >... > d_k = 1" /> seus divisores positivos.

a) Prove que

d_1-d_2+d_3-d_4+...+(-1)^{k-1}d_k=n-1

apenas se n é primo ou n=4.

b) Determine os três inteiros positivos n para os quais

d_1-d_2+d_3-d_4+...+(-1)^{k-1}d_k=n-4.

SOLUÇÃO.

a) Como d_1=n \Rightarrow d_2-d_3+...+(-1)^kd_k=1, mas se k\geq 7 \Rightarrow (d_2-d_3)+(d_4-d_5)+(d_6-d_7)+...+(-1)^k \geq 2, se k=5,6 basta notar que (d_2-d_3)+(d_4-d_5) \geq 2, e para k=4 temos (d_2-d_3)+d_4 \geq 2, logo k=2 ou k=3, se k=2 \Rightarrow n é primo, se k=3 \Rightarrow n=p^2, com p primo e portanto p-1=1 \Rightarrow p=2 \Rightarrow n=4.(Note que (d_i-d_{i+1})\geq 1, para todo i).

b) Se n possuir mais que 10 divisores, então bastar notar que

d_1-d_2+d_3-d_4+...+(-1)^{k-1} \leq n-(d_2-d_3)-(d_4-d_5)-(d_6-d_7)-(d_8-d_9)-(d_{10}-d_{11}) \leq n-5

Logo k\leq 10. Vamos aos casos:

Se k=10, Temos n-(d_2-d_3)-(d_4-d_5)-(d_6-d_7)-(d_8-d_9)-(d_{10})\leq n-5.

Se k=9, devemos ter n da forma n=p^8 ou n=p^2q^2(com p,q primos), note que devemos ter (d_8-1)=1 \Rightarrow d_8=2 \Rightarrow n=2^8, 4q^2 é facil ver que n=256 não é solução e se q\geq 7 \Rightarrow (d_6-d_7)=(q-4)\geq 3, logo q=3,5; mas n=36, 100 não são soluções.

Se k=8, devemos ter n=p^7, p^3q, pqr (com p,q,r primos); se p ou q\geq 5 basta notar que p^2-p\geq 2 e q-p\geq 2, logo n=24, 54, 6r; é fácil ver que n=24, 54 não são soluções. Se n=6r, devemos ter r\leq 7(se não (d_4-d_5)\geq 2), logo n=30, 42; ambos não são soluções.

Se k=7, devemos ter n=p^6, se p\geq 3, temos p^3-p^2\geq 3, além disso, n=2^6=64 não é solução.

Se k=6, devemos ter n=p^5, pq^2; se p\geq 3, como (d_4-d_5)=p^2-p\geq 6 e n=2^5=32 não é solução, temos n=pq^2; se q\geq 3 não é dificil ver que ou (d_2-d_3) ou (d_4-d_5) é maior que 2, logo q=2 \Rightarrow n=4p, logo, ou p=3 \Rightarrow n=12 ou 2p-p+4-2+1=4\Rightarrow p=1(absurdo). Mas note que n=12 é solução.

Se k=5, devemos ter n=p^4, e assim p^3-p^2+p-1=4\Rightarrow p|5 \Rightarrow n=5^4, mas n=625 não é solução.

Se k=4, devemos ter n=pq, p^3; logo, ou q-p+1=4\Rightarrow q-p=3 \Rightarrow (q,p)=(5,2) ou p^2-p+1=4\Rightarrow p|3\Rightarrow p=3, porém n=27 não é solução. Mas n=10 é solução.

Se k=3, devemos ter n=p^2\Rightarrow p-1=4\Rightarrow p=5 e por fim n=25 é solução.

Resposta: n=10, 12, 25.